Difference between revisions of "2018 AMC 10A Problems/Problem 18"

m (Solution 2)
(Solution 3 (Quick Solution))
Line 22: Line 22:
 
Note that the number of total possibilities (ignoring the conditions set by the problem) is <math>3^8=6561</math>. So, E is clearly unrealistic.  
 
Note that the number of total possibilities (ignoring the conditions set by the problem) is <math>3^8=6561</math>. So, E is clearly unrealistic.  
  
Note that if <math>a_7</math> is 1, then it's impossible for <cmath>a_7\cdot3^7+a_6\cdot3^6+a_5\cdot3^5+a_4\cdot3^4+a_3\cdot3^3+a_2\cdot3^2+a_1\cdot3^1+a_0\cdot3^0,</cmath> to be negative. Therefore, if <math>a_7</math> is 1, there are <math>3^7=2187</math> possibilities.
+
Note that if <math>a_7</math> is 1, then it's impossible for <cmath>a_7\cdot3^7+a_6\cdot3^6+a_5\cdot3^5+a_4\cdot3^4+a_3\cdot3^3+a_2\cdot3^2+a_1\cdot3^1+a_0\cdot3^0,</cmath> to be negative. Therefore, if <math>a_7</math> is 1, there are <math>3^7=2187</math> possibilities. (We also must convince ourselves that these <math>2187</math> different sets of coefficients must necessarily yield <math>2187</math> different integer results.)
  
 
As A, B, and C are all less than 2187, the answer must be <math>\boxed{(D) 3281}</math>
 
As A, B, and C are all less than 2187, the answer must be <math>\boxed{(D) 3281}</math>

Revision as of 03:36, 10 February 2018

Problem

How many nonnegative integers can be written in the form \[a_7\cdot3^7+a_6\cdot3^6+a_5\cdot3^5+a_4\cdot3^4+a_3\cdot3^3+a_2\cdot3^2+a_1\cdot3^1+a_0\cdot3^0,\] where $a_i\in \{-1,0,1\}$ for $0\le i \le 7$?

$\textbf{(A) } 512 \qquad  \textbf{(B) } 729 \qquad  \textbf{(C) } 1094 \qquad  \textbf{(D) } 3281 \qquad  \textbf{(E) } 59,048$

Solution

This looks like balanced ternary, in which all the integers with absolute values less than $\frac{3^n}{2}$ are represented in $n$ digits. There are 8 digits. Plugging in 8 into the formula gives a maximum bound of $|x|=3280.5$, which means there are 3280 positive integers, 0, and 3280 negative integers. Since we want all nonnegative integers, there are $3280+1=\boxed{3281}$ integers or $\boxed{D}$.

$QED\blacksquare$

Solution 2

Note that all numbers formed from this sum are either positive, negative or zero. The number of positive numbers formed by this sum is equal to the number of negative numbers formed by this sum, because of symmetry. There is only one way to achieve a sum of zero, if all $a_i=0$. The total number of ways to pick $a_i$ from $i=1, 2, 3, ... 7$ is $3^8=6561$. $\frac{6561-1}{2}=3280$ gives the number of possible negative integers. The question asks for the number of nonnegative integers, so subtracting from the total gives $6561-3280=\boxed{3281}$. (RegularHexagon)

Solution 3 (Quick Solution)

Note that the number of total possibilities (ignoring the conditions set by the problem) is $3^8=6561$. So, E is clearly unrealistic.

Note that if $a_7$ is 1, then it's impossible for \[a_7\cdot3^7+a_6\cdot3^6+a_5\cdot3^5+a_4\cdot3^4+a_3\cdot3^3+a_2\cdot3^2+a_1\cdot3^1+a_0\cdot3^0,\] to be negative. Therefore, if $a_7$ is 1, there are $3^7=2187$ possibilities. (We also must convince ourselves that these $2187$ different sets of coefficients must necessarily yield $2187$ different integer results.)

As A, B, and C are all less than 2187, the answer must be $\boxed{(D) 3281}$

Solution 4

Note that we can do some simple casework: If $a_7=1$, then we can choose anything for the other 7 variables, so this give us $3^7$. If $a_7=0$ and $a_6=1$, then we can choose anything for the other 6 variables, giving us $3^6$. If $a_7=0$, $a_6=0$, and $a_5=1$, then we have $3^5$. Continuing in this vein, we have $3^7+3^6+\cdots+3^1+3^0$ ways to choose the variables' values, except we have to add 1 because we haven't counted the case where all variables are 0. So our total sum is $\boxed{(D) 3281}$. Note that we have counted all possibilities, because the largest positive positive power of 3 must be greater than or equal to the largest negative positive power of 3, for the number to be nonnegative.

See Also

2018 AMC 10A (ProblemsAnswer KeyResources)
Preceded by
Problem 17
Followed by
Problem 19
1 2 3 4 5 6 7 8 9 10 11 12 13 14 15 16 17 18 19 20 21 22 23 24 25
All AMC 10 Problems and Solutions
2018 AMC 12A (ProblemsAnswer KeyResources)
Preceded by
Problem 12
Followed by
Problem 14
1 2 3 4 5 6 7 8 9 10 11 12 13 14 15 16 17 18 19 20 21 22 23 24 25
All AMC 12 Problems and Solutions

The problems on this page are copyrighted by the Mathematical Association of America's American Mathematics Competitions. AMC logo.png